Bashing Telescope

Algebra Level 5

n = 1 24 7 n 3 + 41 n 2 + 88 n + 72 n 5 + 10 n 4 + 35 n 3 + 50 n 2 + 24 n \large\sum_{n=1}^{24}\frac{7n^{3}+41n^{2}+88n+72}{n^{5}+10n^{4}+35n^{3}+50n^{2}+24n} If the value of the above expression is in the form of A B \dfrac{A}{B} , where A , B A,B are positive coprime integers then find the value of A + B A+B .


The answer is 45541.

This section requires Javascript.
You are seeing this because something didn't load right. We suggest you, (a) try refreshing the page, (b) enabling javascript if it is disabled on your browser and, finally, (c) loading the non-javascript version of this page . We're sorry about the hassle.

2 solutions

Shaun Leong
Jan 17, 2016

In the true spirit of bashing, we hope to find the partial fraction form of the expression.

By inspection (rational root theorem helps too), observe that
n 5 + 10 n 4 + 35 n 3 + 50 n 2 + 24 n = n ( n + 1 ) ( n + 2 ) ( n + 3 ) ( n + 4 ) n^5+10n^4+35n^3+50n^2+24n=n(n+1)(n+2)(n+3)(n+4)

Hence let 7 n 3 + 41 n 2 + 88 n + 72 n 5 + 10 n 4 + 35 n 3 + 50 n 2 + 24 n = A n + B n + 1 + C n + 2 + D n + 3 + E n + 4 \frac {7n^3+41n^2+88n+72}{n^5+10n^4+35n^3+50n^2+24n}=\frac {A}{n}+ \frac {B}{n+1} +\frac {C}{n+2} +\frac {D}{n+3}+ \frac {E}{n+4}

Multiplying by the original denominator on both sides, 7 n 3 + 41 n 2 + 88 n + 72 = 7n^3+41n^2+88n+72= A ( n + 1 ) ( n + 2 ) ( n + 3 ) ( n + 4 ) A(n+1)(n+2)(n+3)(n+4) + B n ( n + 2 ) ( n + 3 ) ( n + 4 ) +Bn(n+2)(n+3)(n+4) + C n ( n + 1 ) ( n + 3 ) ( n + 4 ) +Cn(n+1)(n+3)(n+4) + D n ( n + 1 ) ( n + 2 ) ( n + 4 ) +Dn(n+1)(n+2)(n+4) + E n ( n + 1 ) ( n + 2 ) ( n + 3 ) +En(n+1)(n+2)(n+3)

Now, the easiest way is to set n = 0 , 1 , 2 , 3 , 4 n=0,-1,-2,-3,-4 to get a linear equation in terms of each coefficient A,B,C,D and E respectively.

Note: If this were to be a formal proof, we would have to expand and compare coefficients, since by right n cannot be equal to -4,-3,-2,-1,0 otherwise we would have division by zero. Personally I feel this informal method is much faster.

We then obtain ( A , B , C , D , E ) = ( 3 , 3 , 1 , 2 , 3 ) (A,B,C,D,E)=(3,-3,1,2,-3)

7 n 3 + 41 n 2 + 88 n + 72 n 5 + 10 n 4 + 35 n 3 + 50 n 2 + 24 n = 3 n 3 n + 1 + 1 n + 2 + 2 n + 3 3 n + 4 \frac {7n^3+41n^2+88n+72}{n^5+10n^4+35n^3+50n^2+24n}=\frac {3}{n}- \frac {3}{n+1} +\frac {1}{n+2} +\frac {2}{n+3}- \frac {3}{n+4}

Finally n = 1 24 7 n 3 + 41 n 2 + 88 n + 72 n 5 + 10 n 4 + 35 n 3 + 50 n 2 + 24 n \displaystyle \sum_{n=1}^{24}\frac {7n^3+41n^2+88n+72}{n^5+10n^4+35n^3+50n^2+24n} = n = 1 24 ( 3 n 3 n + 1 + 1 n + 2 1 n + 4 + 2 n + 3 2 n + 4 ) =\displaystyle \sum_{n=1}^{24} (\frac {3}{n}- \frac {3}{n+1} +\frac {1}{n+2} - \frac {1}{n+4}+\frac {2}{n+3}- \frac {2}{n+4}) = 3 n = 1 24 ( 1 n 1 n + 1 ) + n = 3 26 ( 1 n 1 n + 2 ) + 2 n = 4 27 ( 1 n 1 n + 1 ) = 3\displaystyle \sum_{n=1}^{24} (\frac {1}{n}-\frac {1}{n+1})+ \displaystyle \sum_{n=3}^{26} (\frac {1}{n}-\frac {1}{n+2})+ 2\displaystyle \sum_{n=4}^{27} (\frac {1}{n}-\frac {1}{n+1}) = 3 ( 1 1 25 ) + ( 1 3 + 1 4 1 27 1 28 ) + 2 ( 1 4 1 28 ) =3(1-\frac {1}{25})+(\frac {1}{3}+ \frac {1}{4}- \frac {1}{27} -\frac {1}{28})+2 (\frac {1}{4}- \frac {1}{28}) = 36091 9450 =\frac {36091}{9450} A + B = 45541 \Rightarrow A+B=\boxed{45541}

Chew-Seong Cheong
Jan 17, 2016

Let the sum be S S , then we have:

S = n = 1 24 7 n 3 + 41 n 2 + 88 n + 72 n 5 + 10 n 4 + 35 n 3 + 50 n 2 + 24 n = n = 1 24 3 ( n 3 + 9 n 2 + 26 n + 24 ) + 4 n ( n 2 + 3 n + 2 ) + 2 n ( n + 1 ) n ( n + 1 ) ( n + 2 ) ( n + 3 ) ( n + 4 ) = n = 1 24 3 ( n + 2 ) ( n + 3 ) ( n + 4 ) + 4 n ( n + 1 ) ( n + 2 ) + 2 n ( n + 1 ) n ( n + 1 ) ( n + 2 ) ( n + 3 ) ( n + 4 ) = n = 1 24 ( 3 n ( n + 1 ) + 4 ( n + 3 ) ( n + 4 ) + 2 ( n + 2 ) ( n + 3 ) ( n + 4 ) ) = n = 1 24 ( 3 [ 1 n 1 n + 1 ] + 4 [ 1 n + 3 1 n + 4 ] + [ 1 n + 2 2 n + 3 + 1 n + 4 ] ) = 3 n = 1 24 1 n 3 n = 1 24 1 n + 1 + n = 1 24 1 n + 2 + 2 n = 1 24 1 n + 3 3 n = 1 24 1 n + 4 = 3 n = 1 24 1 n 3 n = 2 25 1 n + n = 3 26 1 n + 2 n = 4 27 1 n 3 n = 5 28 1 n = 3 1 3 25 + 1 3 + 3 4 1 27 3 28 = 74 25 + 8 27 + 9 14 = 27216 + 2800 + 6075 9450 = 36091 9450 \begin{aligned} S & = \sum_{n=1}^{24} \frac{7n^3+41n^2+88n+72}{n^5+10 n^4 + 35 n^3+50n^2+24n} \\ & = \sum_{n=1}^{24} \frac{3(n^3+9n^2+26n+24)+4n(n^2+3n+2)+ 2n(n+1)}{n(n+1)(n+2)(n+3)(n+4)} \\ & = \sum_{n=1}^{24} \frac{3(n+2)(n+3)(n+4)+4n(n+1)(n+2)+2n(n+1)}{n(n+1)(n+2)(n+3)(n+4)} \\ & = \sum_{n=1}^{24} \left(\frac{3}{n(n+1)} + \frac{4}{(n+3)(n+4)} + \frac{2}{(n+2)(n+3)(n+4)} \right) \\ & = \sum_{n=1}^{24} \left(3 \left[ \frac{1}{n} - \frac{1}{n+1} \right] + 4 \left[ \frac{1}{n+3} - \frac{1}{n+4} \right] + \left[ \frac{1}{n+2} - \frac{2}{n+3} + \frac{1}{n+4} \right] \right) \\ & = 3 \sum_{n=1}^{24} \frac{1}{n} - 3 \sum_{n=1}^{24} \frac{1}{n+1} + \sum_{n=1}^{24} \frac{1}{n+2} + 2 \sum_{n=1}^{24} \frac{1}{n+3} - 3 \sum_{n=1}^{24} \frac{1}{n+4} \\ & = 3 \sum_{n=1}^{24} \frac{1}{n} - 3 \sum_{n=2}^{25} \frac{1}{n} + \sum_{n=3}^{26} \frac{1}{n} + 2 \sum_{n=4}^{27} \frac{1}{n} - 3 \sum_{n=5}^{28} \frac{1}{n} \\ & = \frac{3}{1} - \frac{3}{25} + \frac{1}{3} + \frac{3}{4} - \frac{1}{27} - \frac{3}{28} \\ & = \frac{74}{25} + \frac{8}{27} + \frac{9}{14} \\ & = \frac{27216+2800+6075}{9450} = \frac{36091}{9450} \end{aligned}

A + B = 36091 + 9450 = 45541 \Rightarrow A+B = 36091+9450 = \boxed{45541}

0 pending reports

×

Problem Loading...

Note Loading...

Set Loading...